Please confirm topic selection

Are you sure you want to trigger topic in your Anconeus AI algorithm?

Please confirm action

You are done for today with this topic.

Would you like to start learning session with this topic items scheduled for future?

Review Question - QID 217971

In scope icon L 2
QID 217971 (Type "217971" in App Search)
Figure A is an MRI of a 42-year-old male who presents to your office with complaints of elbow pain. He was recently diagnosed with inflammatory arthritis. He has failed an extensive course of nonoperative management and is interested in proceeding with surgical intervention. On examination, he has relatively preserved motion of the elbow, with pain in the elbow, forearm and wrist. He demonstrates ipsilateral wrist and thumb extension weakness. No instability of the elbow is appreciated. Which of the following would be the most appropriate treatment for this patient?
  • A
  • A
Attach Treatment Poll
Treatment poll is required to gain more useful feedback from members.
Please enter Question Text
Please enter at least 2 unique options
Please enter at least 2 unique options
Please enter at least 2 unique options